The author's argument in the passage would be most weakened if which one of the following were true?

Jazzy on October 8 at 10:10PM

Doesn't the third theory refute B?

If the third theory is correct, then bipedal locomotion would just be developed to avoid threats or find mates. Doesn't that refute B?

Reply
Create a free account to read and take part in forum discussions.

Already have an account? log in

Emil-Kunkin on October 12 at 12:58PM

Was this for a different question? I think this passage is about literature